You are on page 1of 10

RAFFLES INSTITUTION

2013 Year 6 Preliminary Examination Paper 2


H2 Mathematics 9740






No. Solution
1(i)
1
2
: 1 5
2
| |
|
[ =
|
|
\ .
r
and
7 2
: 2 1 , where
8 2
AC
l
| | | |
| |
= + e
| |
| |
\ . \ .
r
C is on
AC
l
7 2
2 for some
8 2
OC

+ | |
|
= + e
|
|
+
\ .


C is on
1
[
2
1 5
2
OC
| |
|
=
|
|
\ .

, i.e.
7 2 2
2 1 5
8 2 2

+ | | | |
| |
+ =
| |
| |
+
\ . \ .


(14 4 ) (2 ) (16 4 ) 5
3

+ + + + + =
=


Hence
7 2 1
2 3 1 = 1
8 2 2
OC
| | | | | |
| | |
=
| | |
| | |
\ . \ . \ .

, i.e. 2 OC = + i j k



(ii)
1 7 6
5 2 3
8 8 0
AB
| | | | | |
| | |
= =
| | |
| | |
\ . \ . \ .

//
2
1
0
| |
|
|
|
\ .
//
2
[ , since
2
[ contains
AB
l

2 1 2
[ [ [ // normal of
1
[ //
2
1
2
| |
|
|
|
\ .


Hence normal of
2
[ //
2 2 2
1 1 4
0 2 4
| | | | | |
| | |
=
| | |
| | |

\ . \ . \ .
//
1
2
2
| |
|
|
|

\ .


Equation of
2
[ is
1 7 1
2 2 2
2 8 2
| | | | | |
| | |
=
| | |
| | |

\ . \ . \ .
r , i.e. ( 2 2 ) 5 + = r i j k

(iii)
Since
2 2
2 1 4 2 2 0
1 2
| | | |
| |
= + + =
| |
| |
\ . \ .
and
2 1
2 2 2 4 2 0
1 2
| | | |
| |
= + =
| |
| |

\ . \ .
,
H2 MA 9740/P2/2013 RI Year 6 Preliminary Examination - Page 2 of 10
2 2 + + i j k is to both normal of
1
[ and normal of
2
[ .

Hence 2 2 + + i j k is parallel to l (verified).


From (i) and (ii), we can deduce that C is on both
1
[ and
2
[ .

Hence the cartesian equation of l is
1 ( 1) 2
2 2 1
x y z
= =

, i.e.
1 1
2
2 2
x y
z
+
= =



No. Solution
2(a)
(i)
































From graph of
2
2 1
2 3
x
y
x x
+
=
+ +
shown above,
f
1
R 1 ,
2
k k
(
= + +
(

.

For gf to be a function,
f g
R D _ .
1 1
2 i.e.
2 2
k k k + < >

(ii)
f
11
R 4,
2
(
=
(


| |
gf
R 0, 1 =

(b) Since h( ) y x = passes through (1,1) and (2, 2) ,
3 2
(1) (1) (1) 1 a b c d + + + = (1)
3 2
(2) (2) (2) 2 a b c d + + + = (2)

Now
2
h ( ) 3 2 x ax bx c ' = + +
Since (2, 2) is a maximum point,
2
3 (2) 2 (2) 0 a b c + + =
12 4 0 a b c + + = (3)


Since ( 4,14) lies on graph of
( )
h y x = ,
either (4,14) or (4, 14) lies on graph of h( ) y x = ,
x
O
y


H2 MA 9740/P2/2013 RI Year 6 Preliminary Examination - Page 3 of 10
i.e. h(4) 14 or 14 = .

As h( ) x is strictly decreasing for 2 x > , h(4) h(2) 2 s = .
Hence h(4) 14 = and
3 2
(4) (4) (4) 14 a b c d + + + =
64 16 4 14 a b c d + + + = (4)


Solving (1) to (4) gives
1 a = , 4 b = , 4 c = , 2 d =



No. Solution
3(i)
2
d
2sin 4sin 8sin cos
d
y
y y u u u u
u
= = =
When
1
2, sin
4 2
y
t
u u = = =
When 4, sin 1
2
y
t
u u = = =

( )
( )
2
4
2
4
2
4
2
4
2
2
2
4 4 4sin
d 8sin cos d
4sin
cos
8sin cos d
sin
8 cos d
y
y
y
t
t
t
t
t
t
u
u u u
u
u
u u u
u
u u

=
=
=
} }
}
}


2
4
2
4
1 cos 2
8 d
2
sin 2
4
2
1
4
2 4 2
2
t
t
t
t
u
u
u
u
t t
t
+
=
(
= +
(

( | |
= +
| (
\ .
=
}



(ii) When 0, 2 x y = =
2
16 16 4
2 4 2 2
( 2) 4
y
y x
x y y

= = =
+

Since 2 x s ,
4
2 2
y
x
y

=
Required area
4
2
d x y =
}

H2 MA 9740/P2/2013 RI Year 6 Preliminary Examination - Page 4 of 10

| |
4
2
4
4
2
2
4
2 1 d
4
2 d
2[2 ( 2)]
2(4 )
y
y
y
y
y y
y
t
t
| |

=
|
|
\ .


=
`

)
=
=
}
}


(iii)
Required volume
2
2 2
0
(4 )(2) d y x t t =
}


2
2
2
0
16
32 d
( 2) 4
35.9 (3 s.f.)
x
x
t t
(
=
(
+

=
}




No. Solution
4(a) Since i z = is a root,
3
i 2i 0 i k k + + = =

Hence the equation becomes
3
2 i 0 z z + = .
3 2
2 i ( i)( 1) z z z z az + = + +
Comparing coefficient of
2
z , i a = .

For
2
i 1 0 z z + + = ,
2
i i 4(1)(1) i 5 i i 5
2(1) 2 2
z

= = =

Hence the other 2 roots are
i+i 5 i i 5
and
2 2

.

(b)
(i)








(ii)









1 1 1
[cos( ) i sin( )] (cos i sin )
w r r
u u u u = + =








500
3 40i
500
(cos i sin ) 3 40i
w
w
r
r
u u
= +
= +




Comparing real and imaginary parts,
H2 MA 9740/P2/2013 RI Year 6 Preliminary Examination - Page 5 of 10















(iii)
500 500
cos 3 , sin 40 r
r r
u u = =
2
3 40
cos , sin
500 500
r r
u u = =

2
2
2
4 2
2 2
2
3 40
1
500 500
9 1600 250000 0
( 100)(9 2500) 0
( 10)( 10)(9 2500) 0
r r
r r
r r
r r r
| |
| |
+ =
| |
\ .
\ .
+ =
+ =
+ + =


Since , 10 r r
+
e =

Subst 10 r = ,
3 4
cos , sin
5 5
u u = =
3 4
10 i 6 8i
5 5
w
| |
= =
|
\ .


Alternative Solution 1:
From
500
3 40i, w
w
= + consider the modulus of both sides to get
2 2
500
3 40i (3 ) 40 . r r
r
= + = + This leads directly to the same equation for r as above, and
is solved similarly to get 10. r = Then
500
6 8i.
3(10) 40i
w = =
+


Alternative Solution 2:
From
2
3 40
cos , sin ,
500 500
r r
u u = = eliminate r to get

( )
2
2 2
16
1 cos .
1
3 500 16
cos sin sin
500 4 15 5 0
u u u u
| |
= =
|
=
\ .

Solve this quadratic to get cos
3
,
5
u = rejecting the other root, and proceed to get
2
500
co 1 0. s 0
3
r u = = Thus 10 r = and
40 4
.
5
in
5
s
00
r u = = Finally
i sin co 6 i. s 8 w r u u + = =

H2 MA 9740/P2/2013 RI Year 6 Preliminary Examination - Page 6 of 10
Section B: Statistics [60 marks]


No. Solution
(i) Obtain the name list of all patients and number them from 1 to N , N being the number of
patients in the clinics records.
Then use a random number generator to obtain 200 numbers from 1 to N , and the patients
with the corresponding numbers will be selected as respondents.

(ii)

Use simple random
sampling to choose y
number of patients from
each age group as
shown in the table, with
the sample size y proportional to the number of patients within each age group.

Age
( x )
20 x s 20 35 x < s

35 50 x < s

50 x >
Sample
size
( y )
15
200
100
30

=

20
200
100
40

=

40
200
100
80

=

25
200
100
50

=

(iii) Simple random sampling may not give a fair representation of patients from different age
groups.



No. Solution
(i)
Number of arrangements
6
12! 6! 2 478955520 = =

(ii)
Number of arrangements
6 5
5
(5 1)! 2 5 4 92160 C = =
Or:
6 5 5
1 2
(5 1)! 2 C P ;
5
6 (6 1)! 2 4 ;
5 6
6 (7 1)! 2 (6 1)! 2 (






No. Solution
(i)

Let
n
R denote the event that a red ball is removed on the
th
n draw.
Let
m
W denote the event that a white ball is removed on the
th
m draw.
1 2
1
P( )
1
r r
R R
r w r w

=
+ +
(or using tree diagram)

2 1 2 1 2
P( ) P( ) P( )
1
1 1
R R R W R
r r w r
r w r w r w r w
r
r w
= +

= +
+ + + +
=
+

1 2
1 2
2
P( ) 1
P( | )
P( ) 1
R R r
R R
R r w

= =
+

H2 MA 9740/P2/2013 RI Year 6 Preliminary Examination - Page 7 of 10
(ii)

1 2 1 2
2
P( ) 1 P( )
1
1
1
2
( )( 1)
W W R R
r r
r w r w
wr w w
r w r w
=

=
+ +
+
=
+ +


(iii) If exactly one white ball remains in the box when the removal stops, ( 1) w white balls must
be removed in the first ( 2) r w + draws.
Required probability
2
( 2)!
1
( 1)!( 1)!
( )!
( )( 1)
! !
r w
r w
w wr w r
r w r w r w r w
w r
w
+ | |
+
|


\ .
= = =
+ + + + | |
|
\ .



No. Solution
(i) To test
0
H : 150 = vs
1
H : 150 < ,
where denotes the population mean lifetime of batteries.

Since p value = 0.0975 > 0.08, we do not reject
0
H and conclude that there is insufficient
evidence, at 8% level of significance, that the population mean lifetime of batteries is less
than 150 hours.

(ii)
10317
t
t
n n
= =


Under
0
H ,
2
0
~ N , T
n
| |
|
\ .
o
approximately by Central Limit Theorem, where
0
150, 16.877 o = = .

value P( ) 0.0975 p T t = s =
10317
150
P 0.0975
16.877
10317
150
1.29593
16.877
10317 21.871
150 0 (shown)
n
Z
n
n
n
n n
| |

|
s = |
|
|
\ .

=
+ =


Solve using GC, 70 n =

(iii) It is NOT necessary to assume that T is normally distributed, since the sample size n is large,
so by Central Limit Theorem, T is approximately normally distributed.





H2 MA 9740/P2/2013 RI Year 6 Preliminary Examination - Page 8 of 10
No. Solution
(i) From GC, 0.916 (3 s.f.) r =
(ii)




(iii) A linear model predicts the average height would increase continuously with no upper bound.
OR:
The scatter diagram does not show the points lie close to a straight line.
OR:
The scatter diagram shows that as x increases, y increases at a decreasing rate.

A quadratic model predicts that the average height reaches a maximum, then decreases with
time and eventually takes on negative values.
(iv) For the suggested model, the least square regression line is
6.11ln 6.10 y x = + .
The product moment correlation coefficient is 0.993 (3 s.f.).

When 5.5, 16.5 x y = =
Hence the average height is 16.5 feet.

Since 5.5 x = is within range of value of x , and the product moment correlation coefficient is
close to 1, suggesting a strong positive linear correlation between the 2 variables, the estimate
is reliable.


No. Solution

(i) ~ N(100 90, 25 36), i.e. ~ N(10, 61) X Y X Y +
( )
P 5 P( 5) P( 5)
0.73897 0.027394
0.766 (3 s.f.)
X Y X Y X Y > = > + <
= +
=

OR:
( ) ( )
P 5 1 P 5 X Y X Y > = <

1 P( 5 5)
1 0.23363
0.766 (3 s.f.)
X Y = < <
=
=


(ii)
2
1 2
1 2
3 ~ N(100 100 3(90), 25+25+3 (36))
i.e. 3 ~ N(470, 374)
X X Y
X X Y
+ + + +
+ +

1 2
P( 3 495) 0.0981 (3 s.f.) X X Y + + > =

Let T be the number of observations, out of 150, for which 90 Y < . Then ~ B(150, 0.5) T .
19.8
6
1
11
x
y
H2 MA 9740/P2/2013 RI Year 6 Preliminary Examination - Page 9 of 10

Since 150 n = is large such that 75 5 np = > and (1 ) 75 5 n p = > , ~ N(75, 37.5) T
approximately.

P( 68) P( 67.5) (by continuity correction)
0.88966 (5 d.p.)
T T > = >
=




No. Solution

(i)

Let M be the number of substandard medium eggs, out of 10.
Then ~ B 10,
100
m
M
| |
|
\ .
.
2 8
10
2
P( 2) 0.3 1 0.3
100 100
m m
M C
| | | |
= = =
| |
\ . \ .

From graphic calculator,
18.6 (3 s.f.) m = or 21.5 (3 s.f.) (rejected since 20 m < )





(ii)

Let X be the number of small eggs, out of 30.
Let Y be the number of large eggs, out of 10.
Then ~ B(30, 0.18) X and ~ B(10, 0.08) Y .

Required probability
P( 5) P(6 7)P( 1)
P( 5) [P( 7) P( 5)][1 P( 0)]
0.53946 (0.84179 0.53946)(1 0.43439)
0.710 (3 s.f.)
X X Y
X X X Y
= s + s s >
= s + s s =
~ +
=



(iii)

Let W be the number of large eggs, out of 60.
Then ~ B(60, 0.08) W .
Since 60 50 n = > is large, 0.08 0.1 p = < is small such that
4.8 5 np = < , ~ Po(4.8) W approximately.

Required probability P( 4) W = s
= 0.476 (3 s.f.)
(iv)

Assume that the daily demand of eggs in the morning and that in the afternoon are
independent.

(v)

Let T be the total daily demand of eggs, in trays of 30.
Then ~ Po(5 19 5 9), i.e. ~ Po(140) T T + .

Let k be the number of trays of eggs the supermarket stocks up each day. We need

H2 MA 9740/P2/2013 RI Year 6 Preliminary Examination - Page 10 of 10
P( ) 0.9 T k s > .

From graphic calculator,
when 154 k = , P( ) 0.88859 0.9 T k s = <
when 155 k = , P( ) 0.90332 0.9 T k s = >
when 156 k = , P( ) 0.91655 0.9 T k s = >

Hence, least value of 155 k = .

You might also like